Covering space of $mathrm{SL}_2(mathbb{R})$












3














How to show that $mathbb{R} times (0,+infty) times mathbb{R}$ is a universal cover of $mathrm{SL}_2(mathbb{R})$ by Iwasawa Decomposition?



My attempt By Iwasawa Decomposition, for any $A in mathrm{SL}_2(mathbb{R})$, we have
$$
A =
begin{pmatrix}
cos theta & -sin theta \
sin theta & cos theta
end{pmatrix}
begin{pmatrix}
r & 0 \
0 & frac 1r
end{pmatrix}
begin{pmatrix}
1 & x \
0 & 1
end{pmatrix}.
$$

Thus we get a map $varphi: mathbb{R} times (0,+infty) times mathbb{R} to mathrm{SL}_2(mathbb{R})$. I think it's sufficient to show that $mathbb{R}$ is a universal cover of the subspace
$$
left{
begin{pmatrix}
cos theta & -sin theta \
sin theta & cos theta
end{pmatrix}
:
theta in mathbb{R}
right}.
$$

But I don't know how to show it rigorously. Can you give me some hints?










share|cite|improve this question




















  • 1




    I don't understand why this was downvoted.
    – Shaun
    Nov 26 at 15:22






  • 5




    the subspace you mention is homeomorphic to a circle (basically, the parametrisation you gave factors into the homeomorphism). Hope that helps.
    – user120527
    Nov 26 at 15:27










  • @user120527 Thanks! It's a good explaination.
    – Kai Xing
    Nov 27 at 1:49
















3














How to show that $mathbb{R} times (0,+infty) times mathbb{R}$ is a universal cover of $mathrm{SL}_2(mathbb{R})$ by Iwasawa Decomposition?



My attempt By Iwasawa Decomposition, for any $A in mathrm{SL}_2(mathbb{R})$, we have
$$
A =
begin{pmatrix}
cos theta & -sin theta \
sin theta & cos theta
end{pmatrix}
begin{pmatrix}
r & 0 \
0 & frac 1r
end{pmatrix}
begin{pmatrix}
1 & x \
0 & 1
end{pmatrix}.
$$

Thus we get a map $varphi: mathbb{R} times (0,+infty) times mathbb{R} to mathrm{SL}_2(mathbb{R})$. I think it's sufficient to show that $mathbb{R}$ is a universal cover of the subspace
$$
left{
begin{pmatrix}
cos theta & -sin theta \
sin theta & cos theta
end{pmatrix}
:
theta in mathbb{R}
right}.
$$

But I don't know how to show it rigorously. Can you give me some hints?










share|cite|improve this question




















  • 1




    I don't understand why this was downvoted.
    – Shaun
    Nov 26 at 15:22






  • 5




    the subspace you mention is homeomorphic to a circle (basically, the parametrisation you gave factors into the homeomorphism). Hope that helps.
    – user120527
    Nov 26 at 15:27










  • @user120527 Thanks! It's a good explaination.
    – Kai Xing
    Nov 27 at 1:49














3












3








3







How to show that $mathbb{R} times (0,+infty) times mathbb{R}$ is a universal cover of $mathrm{SL}_2(mathbb{R})$ by Iwasawa Decomposition?



My attempt By Iwasawa Decomposition, for any $A in mathrm{SL}_2(mathbb{R})$, we have
$$
A =
begin{pmatrix}
cos theta & -sin theta \
sin theta & cos theta
end{pmatrix}
begin{pmatrix}
r & 0 \
0 & frac 1r
end{pmatrix}
begin{pmatrix}
1 & x \
0 & 1
end{pmatrix}.
$$

Thus we get a map $varphi: mathbb{R} times (0,+infty) times mathbb{R} to mathrm{SL}_2(mathbb{R})$. I think it's sufficient to show that $mathbb{R}$ is a universal cover of the subspace
$$
left{
begin{pmatrix}
cos theta & -sin theta \
sin theta & cos theta
end{pmatrix}
:
theta in mathbb{R}
right}.
$$

But I don't know how to show it rigorously. Can you give me some hints?










share|cite|improve this question















How to show that $mathbb{R} times (0,+infty) times mathbb{R}$ is a universal cover of $mathrm{SL}_2(mathbb{R})$ by Iwasawa Decomposition?



My attempt By Iwasawa Decomposition, for any $A in mathrm{SL}_2(mathbb{R})$, we have
$$
A =
begin{pmatrix}
cos theta & -sin theta \
sin theta & cos theta
end{pmatrix}
begin{pmatrix}
r & 0 \
0 & frac 1r
end{pmatrix}
begin{pmatrix}
1 & x \
0 & 1
end{pmatrix}.
$$

Thus we get a map $varphi: mathbb{R} times (0,+infty) times mathbb{R} to mathrm{SL}_2(mathbb{R})$. I think it's sufficient to show that $mathbb{R}$ is a universal cover of the subspace
$$
left{
begin{pmatrix}
cos theta & -sin theta \
sin theta & cos theta
end{pmatrix}
:
theta in mathbb{R}
right}.
$$

But I don't know how to show it rigorously. Can you give me some hints?







abstract-algebra algebraic-topology






share|cite|improve this question















share|cite|improve this question













share|cite|improve this question




share|cite|improve this question








edited Nov 26 at 15:13

























asked Nov 26 at 14:58









Kai Xing

464




464








  • 1




    I don't understand why this was downvoted.
    – Shaun
    Nov 26 at 15:22






  • 5




    the subspace you mention is homeomorphic to a circle (basically, the parametrisation you gave factors into the homeomorphism). Hope that helps.
    – user120527
    Nov 26 at 15:27










  • @user120527 Thanks! It's a good explaination.
    – Kai Xing
    Nov 27 at 1:49














  • 1




    I don't understand why this was downvoted.
    – Shaun
    Nov 26 at 15:22






  • 5




    the subspace you mention is homeomorphic to a circle (basically, the parametrisation you gave factors into the homeomorphism). Hope that helps.
    – user120527
    Nov 26 at 15:27










  • @user120527 Thanks! It's a good explaination.
    – Kai Xing
    Nov 27 at 1:49








1




1




I don't understand why this was downvoted.
– Shaun
Nov 26 at 15:22




I don't understand why this was downvoted.
– Shaun
Nov 26 at 15:22




5




5




the subspace you mention is homeomorphic to a circle (basically, the parametrisation you gave factors into the homeomorphism). Hope that helps.
– user120527
Nov 26 at 15:27




the subspace you mention is homeomorphic to a circle (basically, the parametrisation you gave factors into the homeomorphism). Hope that helps.
– user120527
Nov 26 at 15:27












@user120527 Thanks! It's a good explaination.
– Kai Xing
Nov 27 at 1:49




@user120527 Thanks! It's a good explaination.
– Kai Xing
Nov 27 at 1:49















active

oldest

votes











Your Answer





StackExchange.ifUsing("editor", function () {
return StackExchange.using("mathjaxEditing", function () {
StackExchange.MarkdownEditor.creationCallbacks.add(function (editor, postfix) {
StackExchange.mathjaxEditing.prepareWmdForMathJax(editor, postfix, [["$", "$"], ["\\(","\\)"]]);
});
});
}, "mathjax-editing");

StackExchange.ready(function() {
var channelOptions = {
tags: "".split(" "),
id: "69"
};
initTagRenderer("".split(" "), "".split(" "), channelOptions);

StackExchange.using("externalEditor", function() {
// Have to fire editor after snippets, if snippets enabled
if (StackExchange.settings.snippets.snippetsEnabled) {
StackExchange.using("snippets", function() {
createEditor();
});
}
else {
createEditor();
}
});

function createEditor() {
StackExchange.prepareEditor({
heartbeatType: 'answer',
autoActivateHeartbeat: false,
convertImagesToLinks: true,
noModals: true,
showLowRepImageUploadWarning: true,
reputationToPostImages: 10,
bindNavPrevention: true,
postfix: "",
imageUploader: {
brandingHtml: "Powered by u003ca class="icon-imgur-white" href="https://imgur.com/"u003eu003c/au003e",
contentPolicyHtml: "User contributions licensed under u003ca href="https://creativecommons.org/licenses/by-sa/3.0/"u003ecc by-sa 3.0 with attribution requiredu003c/au003e u003ca href="https://stackoverflow.com/legal/content-policy"u003e(content policy)u003c/au003e",
allowUrls: true
},
noCode: true, onDemand: true,
discardSelector: ".discard-answer"
,immediatelyShowMarkdownHelp:true
});


}
});














draft saved

draft discarded


















StackExchange.ready(
function () {
StackExchange.openid.initPostLogin('.new-post-login', 'https%3a%2f%2fmath.stackexchange.com%2fquestions%2f3014425%2fcovering-space-of-mathrmsl-2-mathbbr%23new-answer', 'question_page');
}
);

Post as a guest















Required, but never shown






























active

oldest

votes













active

oldest

votes









active

oldest

votes






active

oldest

votes
















draft saved

draft discarded




















































Thanks for contributing an answer to Mathematics Stack Exchange!


  • Please be sure to answer the question. Provide details and share your research!

But avoid



  • Asking for help, clarification, or responding to other answers.

  • Making statements based on opinion; back them up with references or personal experience.


Use MathJax to format equations. MathJax reference.


To learn more, see our tips on writing great answers.





Some of your past answers have not been well-received, and you're in danger of being blocked from answering.


Please pay close attention to the following guidance:


  • Please be sure to answer the question. Provide details and share your research!

But avoid



  • Asking for help, clarification, or responding to other answers.

  • Making statements based on opinion; back them up with references or personal experience.


To learn more, see our tips on writing great answers.




draft saved


draft discarded














StackExchange.ready(
function () {
StackExchange.openid.initPostLogin('.new-post-login', 'https%3a%2f%2fmath.stackexchange.com%2fquestions%2f3014425%2fcovering-space-of-mathrmsl-2-mathbbr%23new-answer', 'question_page');
}
);

Post as a guest















Required, but never shown





















































Required, but never shown














Required, but never shown












Required, but never shown







Required, but never shown

































Required, but never shown














Required, but never shown












Required, but never shown







Required, but never shown







Popular posts from this blog

Quarter-circle Tiles

build a pushdown automaton that recognizes the reverse language of a given pushdown automaton?

Mont Emei